The combined math and verbal scores for females taking the SAT-I test are normally distributed with a mean of 998 and a standard deviation of 202 (based on date from the College Board). If a college includes a minimum score of 925 among its requirements, what percentage of females do not satisfy that requirement?

Answers

Answer 1

The percentage of females who do not satisfy the minimum score requirement of 925 on the SAT-I test is 35.9%.

Calculating the z-score for the minimum score requirement:
z = (X - Mean) / Standard Deviation
z = (925 - 998) / 202
z = -73 / 202 ≈ -0.361


Now, using the z-score to find the percentage of females below the minimum score:
Since the z-score is -0.361, we can use a z-table (or an online calculator) to find the area to the left of this z-score, which represents the percentage of females who scored below 925. The area to the left of -0.361 is approximately 0.359.

3. Convert the area to a percentage:
Percentage = Area * 100
Percentage = 0.359 * 100 ≈ 35.9%

So, approximately 35.9% of females do not satisfy the minimum score requirement of 925 on the SAT-I test.

To learn more about mean: https://brainly.com/question/1136789

#SPJ11


Related Questions

Find a set of parametric equations of the line with the given characteristics. (Enter your answers as a comma-separated list.)
The line passes through the point (-4, 8, 7) and is perpendicular to the plane given by -x + 4y + z = 8.

Answers

One possible set of parametric equations for the line is:

x = -4 + 4t
y = 8 - t
z = 7 - 4t

To see why these work, let's first consider the equation of the plane: -x + 4y + z = 8. This can also be written in vector form as:

[ -1, 4, 1 ] · [ x, y, z ] = 8

where · denotes the dot product. This equation says that the normal vector to the plane is [ -1, 4, 1 ], and that any point on the plane satisfies the equation.

Now, since the line we want is perpendicular to the plane, its direction vector must be parallel to the normal vector to the plane. In other words, the direction vector of the line must be some multiple of [ -1, 4, 1 ]. Let's call this direction vector d.

To find d, we can use the fact that the dot product of two perpendicular vectors is zero. So we have:

d · [ -1, 4, 1 ] = 0

Expanding this out, we get:

-1d1 + 4d2 + 1d3 = 0

where d1, d2, d3 are the components of d. This equation tells us that d must be of the form:

d = [ 4k, k, -k ]

where k is any non-zero scalar (i.e. any non-zero real number).

Now we just need to find a point on the line. We're given that the line passes through (-4, 8, 7), so this will be our starting point. Let's call this point P.

We can now write the parametric equations of the line in vector form as:

P + td

where t is any scalar (i.e. any real number). Substituting in the expressions for P and d that we found above, we get:

[ -4, 8, 7 ] + t[ 4k, k, -k ]

Expanding this out, we get the set of parametric equations I gave at the beginning:

x = -4 + 4tk
y = 8 + tk
z = 7 - tk

where k is any non-zero scalar.
To find a set of parametric equations for the line, we first need to determine the direction vector of the line. Since the line is perpendicular to the plane given by -x + 4y + z = 8, we can use the plane's normal vector as the direction vector for the line. The normal vector for the plane can be determined by the coefficients of x, y, and z, which are (-1, 4, 1).

Now that we have the direction vector (-1, 4, 1) and the point the line passes through (-4, 8, 7), we can write the parametric equations as follows:

x(t) = -4 - t
y(t) = 8 + 4t
z(t) = 7 + t

So, the set of parametric equations for the line is {x(t) = -4 - t, y(t) = 8 + 4t, z(t) = 7 + t}.

Learn more about parametric equations here: brainly.com/question/28537985

#SPJ11

what is the probability that a random point on AK will be on BE

Answers

The probability of the event BE falling on a random point AK is 4/11

What is the probability of an event?

A probability event can be defined as a set of outcomes of an experiment. In other words, an event in probability is the subset of the respective sample space.

In this problem, we need to determine our sample space;

The sample space = 11

The number of favorable outcomes = 4

The probability of a random point on AK to be on BE will be;

P = 4 / 11

Learn more on probability here;

https://brainly.com/question/24756209

#SPJ1

Solve the equation. ㏒₃(1/9)=2x-1
Enter your answer in the box. Enter a fractional answer as a simplified fraction.

Answers

The solution to the given equation which is log₃(1/9) = 2x - 1 is equal to x = -1/2.

To solve the equation log₃(1/9) = 2x - 1, we need to isolate the variable x on one side of the equation. We can start by using the logarithm property that states that the logarithm of a number to a base is equal to the exponent to which the base must be raised to obtain that number. In other words, log₃(1/9) = x if and only if [tex]3^x[/tex] = 1/9.

So, let's rewrite the given equation using this property as follows:

[tex]3^{(log(1/9))[/tex] = [tex]3^{2x-1[/tex]

Simplifying the left-hand side using the logarithm property, we get:

1/9 = [tex]3^{(2x - 1)[/tex]

Now, we can solve for x by taking the logarithm of both sides to base 3:

log₃(1/9) = log₃([tex]3^{(2x - 1)[/tex])

-2 = (2x - 1) * log₃(3)

-2 = 2x - 1

2x = -1

x = -1/2

To learn more about logarithm click on,

https://brainly.com/question/23936575

#SPJ1

Jose created a ball pit for his little sister to play in. he put 40 red balls, 55 purple balls, 45 yellow balls, and 60 green balls into the ball pit. while his sister is playing, one ball rolls out of the pit. what is the probability that the ball is red? 0.17 0.17 0.20 0.20 0.25 0.25 0.40

Answers

If he put 40 red balls, 55 purple balls, 45 yellow balls, and 60 green balls into the ball pit. while his sister is playing, one ball rolls out of the pit. Therefore, the probability that the ball that rolled out of the pit is red is 0.2.

The probability of selecting a red ball from the ball pit can be found by dividing the number of red balls by the total number of balls in the pit.

Total number of balls = 40 + 55 + 45 + 60 = 200

Probability of selecting a red ball = Number of red balls / Total number of balls

Probability of selecting a red ball = 40 / 200

Probability of selecting a red ball = 0.2

Therefore, the probability that the ball that rolled out of the pit is red is 0.2.

To learn more about “probability” refer to the https://brainly.com/question/13604758

#SPJ11

Help me pls this is my last try
2. What is the smallest positive degree angle measure equivalent to tan-¹ (0.724)?
42.2°
31.0°
44.6°
35.9°

Answers

You can also use a new

A dealer bought some radios for a total of $1,008. she gave away 6 radios as gifts, sold each of the rest for $14 more than she paid for each radio, and broke even. how many radios did she buy?

Answers

The dealer bought 42 radios.

How many radios did the dealer buy?

Let x be the number of radios the dealer bought.

Let y be the price the dealer paid for each radio.

We know that the dealer bought x radios for a total of $1,008, so:

x * y = 1008

We also know that the dealer gave away 6 radios and sold the rest for $14 more than she paid for each radio, breaking even. This means that the total revenue from selling the remaining radios is equal to the total cost of buying them:

(x - 6) * (y + 14) = x * y

Simplifying this equation, we get:

xy + 14x - 6y - 84 = xy

14x - 6y = 84

7x - 3y = 42 (dividing by 2 on both sides)

Now we have two equations:

x * y = 1008

7x - 3y = 42

We can use substitution or elimination to solve for x and y. Let's use elimination by multiplying the second equation by y/3 and adding it to the first equation:

x * y + (7x - 3y) * (y/3) = 1008 + 42 * (y/3)

xy + 7xy/3 - y²/3 = 1008 + 14y

10xy/3 - y²/3 - 14y - 1008 = 0

Multiplying both sides by 3, we get:

10xy - y² - 42y - 3024 = 0

Now we can use the quadratic formula to solve for y:

y = (-b ± sqrt(b² - 4ac)) / 2a

where a = -1, b = -42, and c = -3024:

y = (-(-42) ± sqrt((-42)² - 4(-1)(-3024))) / 2(-1)

y = (42 ± sqrt(42² - 4*3024)) / 2

y = (42 ± 126) / 2

y = 84 or y = -42

Since the price of a radio cannot be negative, we can discard the second solution and conclude that y = 84.

Now we can solve for x using the first equation:

x * y = 1008

x * 84 = 1008

x = 12

Therefore, the dealer bought 12 radios.

Learn more about radios

brainly.com/question/866212

#SPJ11

Penelope invested $89,000 in an account paying an interest rate of 6 1/4% compounded continuously. Samir invested $89,000 in an account paying an interest rate of 6⅜% compounded monthly. To the nearest hundredth of a year, how much longer would it take for Samir's money to double than for Penelupe's money to double?

Answers

Answer: -10.57

Step-by-step explanation:

Answer:

0.25 years

Step-by-step explanation:

Penelope invested $89,000 in an account paying an interest rate of 6⅜% compounded continuously.

To calculate the time it would take Penelope's money to double, use the continuous compounding interest formula.

[tex]\boxed{\begin{minipage}{8.5 cm}\underline{Continuous Compounding Interest Formula}\\\\$ A=Pe^{rt}$\\\\where:\\\\ \phantom{ww}$\bullet$ $A =$ final amount \\\phantom{ww}$\bullet$ $P =$ principal amount \\\phantom{ww}$\bullet$ $e =$ Euler's number (constant) \\\phantom{ww}$\bullet$ $r =$ annual interest rate (in decimal form) \\\phantom{ww}$\bullet$ $t =$ time (in years) \\\end{minipage}}[/tex]

As the principal amount is doubled, then A = 2P.

Given interest rate:

r = 6.375% = 0.06375

Substitute A = 2P and r = 0.06375 into the continuous compounding interest formula and solve for t:

[tex]\implies 2P=Pe^{0.06375t}[/tex]

[tex]\implies 2=e^{0.06375t}[/tex]

[tex]\implies \ln 2=\ln e^{0.06375t}[/tex]

[tex]\implies \ln 2=0.06375t\ln e[/tex]

[tex]\implies \ln 2=0.06375t(1)[/tex]

[tex]\implies \ln 2=0.06375t[/tex]

[tex]\implies t=\dfrac{\ln 2}{0.06375}[/tex]

[tex]\implies t=10.872896949...[/tex]

Therefore, it will take 10.87 years for Penelope's investment to double.

[tex]\hrulefill[/tex]

Samir invested $89,000 in an account paying an interest rate of 6¹/₄% compounded monthly.

To calculate the time it would take Samir's money to double, use the compound interest formula.

[tex]\boxed{\begin{minipage}{8.5 cm}\underline{Compound Interest Formula}\\\\$ A=P\left(1+\frac{r}{n}\right)^{nt}$\\\\where:\\\\ \phantom{ww}$\bullet$ $A =$ final amount \\ \phantom{ww}$\bullet$ $P =$ principal amount \\ \phantom{ww}$\bullet$ $r =$ interest rate (in decimal form) \\ \phantom{ww}$\bullet$ $n =$ number of times interest is applied per year \\ \phantom{ww}$\bullet$ $t =$ time (in years) \\ \end{minipage}}[/tex]

As the principal amount is doubled, then A = 2P.

Given values:

A = 2PP = Pr = 6.25% = 0.0625n = 12 (monthly)

Substitute the values into the formula and solve for t:

[tex]\implies 2P=P\left(1+\dfrac{0.0625}{12}\right)^{12t}[/tex]

[tex]\implies 2=\left(1+\dfrac{0.0625}{12}\right)^{12t}[/tex]

[tex]\implies 2=\left(1+0.005208333...\right)^{12t}[/tex]

[tex]\implies 2=\left(1.005208333...\right)^{12t}[/tex]

[tex]\implies \ln 2=\ln \left(1.005208333...\right)^{12t}[/tex]

[tex]\implies \ln 2=12t \ln \left(1.005208333...\right)[/tex]

[tex]\implies t=\dfrac{\ln 2}{12 \ln \left(1.005208333...\right)}[/tex]

[tex]\implies t=11.1192110...[/tex]

Therefore, it will take 11.12 years for Samir's investment to double.

[tex]\hrulefill[/tex]

To calculate how much longer it would take for Samir's money to double than for Penelope's money to double, subtract the value of t for Penelope from the value of t for Samir:

[tex]\begin{aligned}\implies t_{\sf Samir}-t_{\sf Penelope}&=11.1192110......-10.872896949...\\&= 0.246314066...\\&=0.25\; \sf years\;(nearest\;hundredth)\end{aligned}[/tex]

Therefore, it would take 0.25 years longer for Samir's money to double than for Penelope's money to double.

PYTHAGOREAN THEOREM!! HELP!! BRAINLIEST!! 20 POINTS!!


I know A and B! I need help with the rest!


Part A

The Pythagorean Theorem states that for any given right triangle, a^2+ b^2 = c^2.

Using the Pythagorean Theorem, what would be the relationship between the areas of the three squares (1, 2,and 3)?


Part B

Using squares 1, 2, and 3, and eight copies of the original triangle, you can create squares 4 and 5. What are the side lengths of square 4 and square 5 in terms of a and b? Do the two squares have the same area?


Part C

Write an expression for the area of square 4 by combining the areas of the four triangles and the two squares.


Part D

Write an expression for the area of square 5 by combining the area of the four triangles and one square.


Part E

Since the areas of square 4 and square 5 are the same, set the two expressions equal.


Part F

Which term is on both sides of the equal sign? Since it’s on both sides of the equal sign, you can cancel it out. What is the expression after canceling out the common term?


Part G

What does the equation show after you cancel out a common term?

Answers

The relationship between the areas of the three squares is that square A plus square B equals the area of square C.

What is Pythagorean Theory?

The Pythagorean theorem is a fundamental idea in geometry that states that for any right-angled triangle, the square of the length of the longest side (opposite the right angle) is equal to the sum of the square of the lengths of the two remaining sides. This equation can be expressed as:

[tex]a^2 + b^2 = c^2[/tex]

Thus, the relationship between the areas of the three squares is that square A plus square B equals the area of square C.

Read more about Pythagorean Theory here:

https://brainly.com/question/343682

#SPJ1

everyone pls answer the questions I posted they are urgent​

Answers

Answer:

unfortunately there's no questions to be answered

Suppose a 4 is rolled on a number cube with sides numbered 1, 2, 3, 4, 5, and 6. The
complement of this event would be rolling a 1, 2, 3, 5, or 6. What is the probability of the
complement, written as a fraction in simplest form?

Answers

The probability of rolling any number other than 4 on a number cube with sides numbered 1, 2, 3, 4, 5, and 6 is 5/6, which can be written as a fraction in simplest form.

The complement of rolling a 4 on a number cube with sides numbered 1, 2, 3, 4, 5, and 6 is rolling any number other than 4, which includes rolling a 1, 2, 3, 5, or 6.

To find the probability of the complement, we need to add up the probabilities of rolling each of these numbers.

Since each number has an equal chance of being rolled, we can find the probability of rolling each number by dividing 1 by the total number of possible outcomes (which is 6, since there are six sides on the cube).

Then, we can add up the probabilities of rolling each of the five numbers in the complement:

P(rolling a 1, 2, 3, 5, or 6) = P(rolling a 1) + P(rolling a 2) + P(rolling a 3) + P(rolling a 5) + P(rolling a 6)

P(rolling any number other than 4) = 1 - P(rolling a 4)

P(rolling any number other than 4) = 1 - 1/6 = 5/6

Therefore, the probability of rolling any number other than 4 on a number cube with sides numbered 1, 2, 3, 4, 5, and 6 is 5/6, which can be written as a fraction in simplest form.

Read more about probabilities at:

brainly.com/question/11234923

#SPJ1

Help with geometry on equations of circles. What would RSQ be?

Answers

Answer:

  34.8°

Step-by-step explanation:

You want the angle between a tangent and a segment to the center from a point on the tangent that is 6 units from the circle of radius 8 units.

Sine

The trig relation useful here is ...

  Sin = Opposite/Hypotenuse

  sin(S) = RQ/SQ

The length QT is the same as QR, so we have ...

  sin(S) = 8/(8 +6)

  S = arcsin(8/(8+6)) ≈ 34.8°

Pls help me with this-

Answers

The formula for the function h(x) is given as follows:

h(x) = g(x + 5).

What is a translation?

A translation happens when either a figure or a function is moved horizontally or vertically on the coordinate plane.

The four translation rules for functions are defined as follows:

Translation left a units: f(x + a).Translation right a units: f(x - a).Translation up a units: f(x) + a.Translation down a units: f(x) - a.

The function h(x) is a translation left 5 units of the function g(x), hence it is defined as follows:

h(x) = g(x + 5).

More can be learned about translations at brainly.com/question/28174785

#SPJ1

A model car is drawn at a scale of 21 to 1. If the model car is 9. 2in. Long, how long is the actual car in feet?

Answers

A model car is drawn at a scale of 21 to 1. If the model car is 9. 2in.  The length of the actual car in feet is approximately 0.7665 feet.

Find out the length of the actual car in feet, we need to first convert the length of the model car from inches to feet.
9.2 inches = 0.767 feet (divide by 12 since there are 12 inches in a foot)
Now, we can use the scale of 21 to 1 to find the length of the actual car in feet.
21 units on the model car = 1 unit on the actual car
So,
1 unit on the actual car = 0.767 feet / 21 = 0.0365 feet
Find the length of the actual car, we can multiply the scale ratio by the length of the model car in units:
21 units x 0.0365 feet per unit = 0.7665 feet
Therefore, the length of the actual car in feet is approximately 0.7665 feet.

Read more about length of the actual car.

https://brainly.com/question/21632956

#SPJ11

The actual car is  0.7665 feet long.

First, we need to convert the length of the model car from inches to feet:

9.2 in. = 9.2/12 ft. = 0.7667 ft.

Next, we can use the scale to find the length of the actual car:

21 units on the drawing = 1 unit in real life

So, we have:

1 unit in real life = length of actual car

21 units on the drawing = length of model car

Substituting the values we have:

1 unit in real life = (0.7667 ft.)/21 = 0.0365 ft.

Therefore, the length of the actual car is:

1 unit in real life x 21 = 0.0365 ft. x 21 = 0.7665 ft.

So, the actual car is approximately 0.7665 feet long.

Learn more about scale at https://brainly.com/question/28521683

#SPJ11

Ava has two frogs. This is __
1
3 the number of frogs that Heather
has. How many frogs does Heather have? Draw a diagram to
represent the division. Then write and solve an equation.

Answers

The value of n which is the number of frogs Heather has is 6.

What is the number of frogs Heather has?

The number of frogs Heather has is calculated as follows;

let the number of frogs Heather has = n

So Ava has 2 fogs, which is equal to 1/3 n.

The value of n which is the number of frogs Heather has is calculated as follows;

(1/3) n = 2

multiply both sides by 3;

n = 3 x 2

n = 6

The division using a diagram, is determined as;

    0         0

    I           I

    I           I

    I            I

Learn more about divisions here: https://brainly.com/question/25289437

#SPJ1

The cost C (in dollars) for the care and maintenance of a horse and carriage is C=15x+2000, where x is the number of rides. Write an equation for the revenue R in terms of the number of rides.

Answers

The equation for revenue R in terms of the number of rides x is given by R = px, where p is the amount charged per ride (in dollars).

The equation for the revenue R in terms of the number of rides can be derived by multiplying the number of rides with the amount charged per ride.

Let the amount charged per ride be p (in dollars).

Then, the equation for revenue R can be written as R = px.

Note that the amount charged per ride is not given in the problem. It can be assumed that the amount charged is a fixed amount for all the rides.

However, the equation for revenue can still be written in terms of the variable p as R = px.

To know more about revenue, refer here:

https://brainly.com/question/29148322#

#SPJ11

How much greater is


f(4) than g (4) if f(x) Is exponential and g (x) is linear?

Answers

When comparing the values of f(4) and g(4), we need to take into account the fact that f(x) is exponential and g(x) is linear. Exponential functions grow at an increasing rate as x increases, while linear functions grow at a constant rate. Therefore, as x gets larger, the difference between f(x) and g(x) will become greater.

To find out how much greater f(4) is than g(4), we first need to calculate the values of f(4) and g(4). Let's say that f(x) = 2^x and g(x) = 3x + 1. Plugging in x = 4, we get:
f(4) = 2^4 = 16
g(4) = 3(4) + 1 = 13

So, f(4) is greater than g(4) by a difference of 3. However, this does not take into account the fact that f(x) is exponential and g(x) is linear.

To see the impact of the different growth rates, let's compare the values of f(x) and g(x) for a range of values of x. We can create a table to compare the two functions:

x    f(x)    g(x)
0    1       1
1    2       4
2    4       7
3    8       10
4    16      13
5    32      16

From this table, we can see that as x increases, the difference between f(x) and g(x) grows at an increasing rate. This is because f(x) is growing exponentially, while g(x) is growing linearly.

In summary, f(4) is 16 and g(4) is 13, so f(4) is greater than g(4) by a difference of 3. However, we also need to take into account the fact that f(x) is exponential and g(x) is linear. As x increases, the difference between f(x) and g(x) will grow at an increasing rate. Therefore, the difference between f(4) and g(4) is not only 3, but also growing exponentially.

To know more about greater refer here

https://brainly.com/question/30707235#

#SPJ11

Polly bought 50 necklaces for £5 each. She sold all the necklaces and made a 70% profit on the original cost. Polly sold 40% of the necklaces for £11 each. 1 She then reduced the price and sold 3 of the remaining necklaces for £8 each. She sold all the remaining necklaces for the same price. Work out this price. ​

Answers

If Polly reduced the price and sold 3 of the remaining necklaces for £8 each, she sold the remaining necklaces for £6.70 each.

First, let's find the original cost of the necklaces:
50 necklaces * £5 = £250

Now, let's calculate the profit Polly made:
£250 * 70% = £175

So, the total amount she made from selling the necklaces is:
£250 + £175 = £425

Polly sold 40% of the necklaces for £11 each:
50 necklaces * 40% = 20 necklaces
20 necklaces * £11 = £220

She sold 3 necklaces for £8 each:
3 necklaces * £8 = £24

Now let's find the amount left after selling these necklaces:
£425 - £220 - £24 = £181

Polly has 50 - 20 - 3 = 27 necklaces remaining. Let's find the price at which she sold each of the remaining necklaces:
£181 / 27 = £6.70

So, Polly sold the remaining necklaces for £6.70 each.

More on price: https://brainly.com/question/19735564

#SPJ11

‼️WILL MARK BRAINLIEST‼️

Answers

The true statements are:

The range for the African-American outline is greater than the range for the Holocaust outline, so there is more variability in the data set for African-American outline.The mean for the Holocaust outline, 38.75, is greater than the mean for the African- American outline, 35, so a student working on the Holocaust project spent more time on the outline on average than a student working on the African-American project.

A valid conclusion is :

The times for the Holocaust outline were greater but less variable.

What is the range and mean of a data set?

The range of a data set is the difference between the highest and lowest values in the set.

To calculate the range, you subtract the smallest value from the largest value.

The mean of a data set, also called the average, is the sum of all the values in the set divided by the total number of values in the set.

To calculate the mean, you add up all the values and divide by the total number of values.

Learn more about the range and mean at: https://brainly.com/question/30094086

#SPJ1

Weights of erasers produced by a certain factory are known to follow the uniform distribution between 31. 5 g and 32. 3 g.

(a) (10 points) erasers produced by this factory are sold in packs of 45. A retailer randomly bought 200 packs. Find the probability that, for at least 15 packs, the average weight of the erasers in the pack is at least 31. 95 g.

(b) (10 points) each day, a quality control unit examines the erasers produced by this factory. The unit randomly chooses an eraser from the outputs of this factory and weighs it. This process is repeated 50 times. The unit then records the total number of erasers that were found to weigh at least 31. 7 g. (erasers with weights at least 31. 7 g are called "good" erasers)suppose this unit works for 42 consecutive days. Find the probability that, on average, it finds at least 37. 2 "good" erasers per day

Answers

a) The probability that, for at least 15 packs, the average weight of the erasers in the pack is at least 31.95 g is approximately 0.0384.

b) The probability that, on average, the unit finds at least 37.2 "good" erasers per day is approximately 0.3133.

a) To solve this problem, we need to use the central limit theorem. According to this theorem, the distribution of sample means becomes approximately normal, regardless of the shape of the population distribution, when the sample size is sufficiently large (usually, n >= 30). In this case, since the sample size is 45, we can assume that the distribution of sample means will be approximately normal.

Now, we need to find the probability that the average weight of at least 15 packs is at least 31.95 g. We can use the normal distribution to calculate this probability. We first calculate the z-score for this value as follows:

z = (31.95 - 31.9) / (0.163 / √(45)) = 1.77

Using a standard normal table or calculator, we can find the probability that a z-score is greater than or equal to 1.77. This probability is approximately 0.0384.

b) To solve this problem, we need to use the normal approximation to the binomial distribution. Since each eraser is either "good" or "bad", the number of "good" erasers that the unit finds each day follows a binomial distribution with parameters n = 50 and p = probability of finding a "good" eraser = (32.3 - 31.7)/(32.3 - 31.5) = 0.5.

Now, we need to find the probability that, on average, the unit finds at least 37.2 "good" erasers per day. We can use the normal distribution to calculate this probability. We first calculate the z-score for this value as follows:

z = (37.2 - 25) / 25 = 0.488

Using a standard normal table or calculator, we can find the probability that a z-score is greater than or equal to 0.488. This probability is approximately 0.3133.

To know more about probability here

https://brainly.com/question/11234923

#SPJ4

It takes Alex 22 minutes to walk from his home to the store. The function (x) - 2. 5x models the distance that Alex has walked in x minutes after leaving his house


to go to the store. What is the most appropriate domain of the function?

Answers

The most appropriate domain of the function is 0 ≤ x ≤ 22. This is because Alex can only walk from his home to the store within a maximum of 22 minutes, and the distance he walks can only be modeled within that time frame.

It is given that the function f(x) = 2.5x, which models the distance Alex walks in x minutes after leaving his house to go to the store. It takes him 22 minutes to walk from his home to the store. The most appropriate domain of the function is the range of x values that make sense in this context.

Step 1: Identify the minimum and maximum values for x.
In this case, the minimum value for x is when Alex starts walking, which is 0 minutes. The maximum value for x is when he reaches the store, which is 22 minutes.

Step 2: Express the domain as an interval.
The domain of the function can be written as an interval from the minimum to the maximum value, including both endpoints. Therefore, the domain is [0, 22].

Therefore, the most appropriate domain of the function f(x) = 2.5x, which models the distance Alex walks in x minutes after leaving his house to go to the store, is [0, 22].

To know more about the domain of the function refer here:

https://brainly.com/question/13113489

#SPJ11

Two liters of the Gatorade cost $3.98. How much do 8 liters cost?

Answers

Answer:

$15.92

Step-by-step explanation:

We Know

2 liters of Gatorade cost $3.98

How much do 8 liters cost?

We take

3.98 x 4 = $15.92

So, 8 liters cost $15.92

Prove that triangle FGH is right-angled at F

Answers

Triangle FGH is a right triangle because (HG)²= (FG)²+ (FH)²

What are similar triangles?

Similar triangles are triangles that have the same shape, but their sizes may vary. The ratio of corresponding sides of similar triangles are equal.

Therefore;

6/5 = 3.6/FH

represent FH by x

6/5 = 3.6/x

6x = 5 × 3.6

6x = 18

divide both sides by 6

x = 18/6 = 3

Since FH is 3, this means that the sides of triangle FGH are Pythagorean triple, hence FGH is a right triangle.

learn more about similar triangles from

https://brainly.com/question/14285697

#SPJ1

Find the area under the standard normal distribution curve between z=0 and z=0. 98

Answers

The area under the standard normal distribution curve between z = 0 and z = 0.98 is:

                         0.8365 - 0.5000 = 0.3365

To find the area under the standard normal distribution curve between z = 0 and z = 0.98, we can use a standard normal distribution table or a calculator that can compute normal probabilities.

Using a standard normal distribution table, we can look up the area corresponding to a z-score of 0 and a z-score of 0.98 separately and then subtract the two areas to find the area between them.

The area under the standard normal distribution curve to the left of z = 0 is 0.5000 (by definition). The area under the curve to the left of z = 0.98 is 0.8365 (from the standard normal distribution table).

So the area under the standard normal distribution curve between z=0 and z=0.98 is approximately 0.3365.

To know more about area under curve refer here

https://brainly.com/question/40445978#

#SPJ11

If the sides of a rectangle are in the ratio 3:4 and the length of the diagonal is 10 cm, find the length of the sides

Answers

Answer:

if the diagonal is 10 then the sides are 3*2 and 4*2 which is 6 and 8 respectively because the diagonal makes it a right angled triangle whereby the the 3,4,5 line steps in, so if the diagonal(hypotenuse) is 10 the 10/5 is 2 then you multiply both 3 and 4 by 2 and that gives you the length of two sides

x^2+8x+16 What is the perfect factored square trinomial

Answers

Answer:

The perfect factored square trinomial that is equivalent to the expression x^2 + 8x + 16 is:

(x + 4)^2

To see why this is the case, you can expand the expression (x + 4)^2 using the FOIL method:

(x + 4)^2 = (x + 4) * (x + 4)

= x^2 + 4x + 4x + 16

= x^2 + 8x + 16

So, x^2 + 8x + 16 can be factored as (x + 4)^2, which is a perfect square trinomial.

Credit card payment terms. paul's credit card closes on the 6th of the month, and his payment is due on paul’s credit card closes on the 6th of the month, and his payment is due on the 24th. if paul purchases a stereo for $300 on june 8th,


how many interest-free days will he have? when will he have to pay for the stereo in full in order to avoid finance charges? (hint: assume that paul pays off his credit


card each month.)


if paul purchases a stereo for $300 on june 8th, the number of interest-free days he will have is i. (round to the nearest whole number.)

Answers

Paul has 18 interest-free days for the $300 stereo purchase.

He will need to pay the full balance of his June billing statement.

If Paul's credit card closes on the 6th of the month and his payment is due on the 24th, then he has 18 days between the close of the billing cycle and the due date of his payment.

If Paul purchases a stereo for $300 on June 8th, then the transaction will be included in his billing cycle for the month of June. Since his billing cycle closes on the 6th, the $300 charge will appear on his June billing statement.

If Paul pays off his credit card in full each month, then he will need to pay the full balance of his June billing statement by the due date of June 24th to avoid finance charges. This means he will need to pay $300 for the stereo, plus any other charges that may have been included on his billing statement for the month of June.

Therefore, the number of interest-free days that Paul will have for the $300 stereo purchase is 18 days, which is the number of days between the billing cycle close date (June 6th) and the payment due date (June 24th).

To summarize:

Paul has 18 interest-free days for the $300 stereo purchase.

Paul will need to pay the full balance of his June billing statement, including the $300 stereo charge, by June 24th to avoid finance charges.

To know more about credit card payment, refer to the link below:

https://brainly.com/question/27333734#

#SPJ11

What would be a theoretical antidote and prescription for Zombies Epsilon, Zeta and Eta?


Zombie Epsilon


Zombie Zeta Zombre Eta


Strand


3. 5


7. 1


e


Amount of Virus (mag/ml) 150 230,636


62


Equation


Days (Doses Needed)


e days


Lays


41 days

Answers

Zombie Epsilon would require 52.5 days of doses, Zombie Zeta would need 163.3 days, and Zombie Eta would require 636e days to be cured.

To develop a theoretical antidote, you would need to consider the virus strand, concentration (mag/ml), and the equation to calculate the number of doses needed.

For Zombie Epsilon, Zeta, and Eta, the amounts of virus are 150, 230, and 636 mag/ml, respectively. To create an effective antidote, you would need to identify the specific virus strands for each zombie type (e.g., strand 3.5 for Epsilon, 7.1 for Zeta, and "e" for Eta).

Using the provided information, the equation should be used to determine the number of days (doses needed) for each zombie type. As an example, let's assume the equation is as follows: Days = (Amount of Virus * Strand) / 10.

For Zombie Epsilon: Days = (150 * 3.5) / 10 = 52.5 days
For Zombie Zeta: Days = (230 * 7.1) / 10 = 163.3 days
For Zombie Eta: Days = (636 * e) / 10 = 636e days (where e is a constant value)

In this theoretical scenario, Zombie Epsilon would require 52.5 days of doses, Zombie Zeta would need 163.3 days, and Zombie Eta would require 636e days to be cured.

Please note that this is a fictional scenario and not based on real-life medical information.

To learn more about equation, refer below:

https://brainly.com/question/29657983

#SPJ11

It takes a boat hr to go 12 mi downstream, and 6 hr to return. Find the rate of the boat in still water and the rate of the current

Answers

The rate of the boat in still water is 5 miles per hour and rate of the boat in current is 3 miles per hour.

Let us represent the rate of boat in still water hence and rate of boat in current be y. Also, we know that speed = distance/time. Hence, keep the values in formula -

Converting mixed fraction to fraction, time = 3/2 hour

Time = 1.5 hour

1.5 (x + y) = 12 : equation 1

Divide the equation 1 by 3

0.5 (x + y) = 4 : equation 2

6 (x - y) = 12 : equation 3

Divide the equation 3 by 6

(x - y) = 2

x = 2 + y : equation 4

Keep the value of x from equation 4 in equation 2

0.5 (2 + y + y) = 4

1 + y = 4

y = 4 - 1

y = 3 miles/ hour

Keep the value y in equation 4 to get x

x = 2 + 3

x = 5 miles per hour

The rate in still water and current is 5 and 3 miles per hour.

Learn more about rate -

https://brainly.com/question/4800946

#SPJ4

The complete question is-

It takes a boat 1 (1/2) hr to go 12 mi downstream, and 6 hr to return. Find the rate of the boat in still water and the rate of the current.

An electronics retailer offers an optional protection plan for a mobile phone it sells. Customers can choose to buy the protection plan for \$100$100dollar sign, 100, and in case of an accident, the customer pays a \$50$50dollar sign, 50 deductible and the retailer will cover the rest of the cost of that repair. The typical cost to the retailer is \$200$200dollar sign, 200 per repair, and the plan covers a maximum of 333 repairs.

Let X be the number of repairs a randomly chosen customer uses under the protection plan, and let F be the retailer's profit from one of these protection plans. Based on data from all of its customers, here are the probability distributions of X and F:

X=\# \text{ of repairs}X=# of repairsX, equals, \#, start text, space, o, f, space, r, e, p, a, i, r, s, end text 000 111 222 333

F=\text{ retailer profit}F= retailer profitF, equals, start text, space, r, e, t, a, i, l, e, r, space, p, r, o, f, i, t, end text \$100$100dollar sign, 100 -\$50−$50minus, dollar sign, 50 -\$200−$200minus, dollar sign, 200 -\$350−$350minus, dollar sign, 350

Probability 0. 900. 900, point, 90 0. 70. 070, point, 07 0. 20. 020, point, 02 0. 10. 010, point, 01

Find the expected value of the retailer's profit per protection plan sold

Answers

Note that the expected value of the retailers profit is - $114. This means he made a loss.

How did we arrive at this ?

To find the expected value we must proceed as follows

Expected Value - E(F) is

Probability of F - P(F)

= 100 x ($100 - $200) + (P(F) = $50) x ($50 - $200) + (P(F) = $ -200) x ( $ - 200 - $200) +  (P(F)   = $- 350) x ($ -350  $ 200)

= (0.9) x (-100) + (0.07 ) x (-150) + (0.01)  x  (-550)  + (0.02) x (-400)

= - 90  - 10.5  - 5.5 -8

E(F) = $ -114

So it is right to state that the expected value of the retailer's profit per protection plan sold is -$114, which  is a loss.

Learn more about expected value:
https://brainly.com/question/29574962
#SPJ4

Full Question:

An electronics retailer sells mobile phones with an optional protection plan for $100. In case of an accident, the customer pays a $50 deductible and the retailer covers the rest of the repair cost, which is typically $200 per repair. The protection plan covers a maximum of 333 repairs.

Let X be the number of repairs a randomly chosen customer uses under the protection plan, and let F be the retailer's profit from one of these protection plans. The probability distributions of X and F are:

X = number of repairs: 0 1 2 3

Probability: 0.90 0.07 0.02 0.01

F = retailer profit: $100-$50-$200-$350

Probability: 0.90 0.07 0.02 0.01

The task is to find the expected value of the retailer's profit per protection plan sold.

Lokota wants to build a sandbox for his little brother. Determine the amount of sand he needs by finding the area of the sandbox. Use the drop-down menus to complete the statements.



First, write the
.



Next, use parentheses when you substitute


for b and


for h.



Now, simplify by


1


2


, 2. 4, and 3. 5.



The area of the sandbox is


Answers

The area of the sandbox whose base is 3.5 meter and height is 2.4 meter is 4.2 m².

Given:

Base = 3.5 m

Height = 2.4 m

First, the area of the sandbox formula:

Area = 1/2 x base x height.

Next,  substitute b = 3.5 meters and h = 2.4 meters.

Area = 1/2 * (3.5) * (2.4).

Now, simplify by multiplying 1/2, 2.4, and 3.5.

Area = 1/2 x 2.4 x 3.5

Area = 4.2 square meters.

Thus, The area of the sandbox is 4.2 m².

Learn more about area here:

https://brainly.com/question/30307509

#SPJ12

The question attached here seems to be incomplete, the complete question is:

Lokota wants to build a sandbox for his little brother. Determine the amount of sand he needs by finding the area of the sandbox. Use the drop-down menus to complete the statements.

First, write the formula: A = 1/2 bh
Next, use parentheses when you substitute __ for b and __ for h.

Now, simplify by ___ 1/2, 2.4, and 3.5.

The area of the sandbox is ___ m²

Other Questions
There are 90 children in year 6 at woodland junior schoolthey are split into three classesclassnumber n class276m6p336t30each child chose football or netball or hockey.in 6m, 13 children chose hockey.the rest of the class were split equally between football and netball.in 6p, 9 children chose netballtwice as many children chose football as chose hockeyin 6t the ratio of children who chosefootball to netball to hockey was 1:2:3complete this tableclassnumber in classfootballnetballhockey627136p336t30 If the electric potential at point A in the electric field created bya point charge of 3. 3 * 10-11 C is 0. 6 V, what's the distancebetween point A and the point charge? Estimate kas 9. 00 x109Nm2/C2 EXAMPLE 1 A spring with a mass of 6 kg has a natural length of 0.3 m. A force of 38.4 N is required to maintain it stretched to a length of 0.7 m. If the spring is stretched to a length of 0.7 m and then released with initial velocity 0, find the position of the mass at time t. SOLUTION From Hooke's Law, the force required to stretch the spring is k(0.4) = 38.4 so k = 38.4/0.4 = 96. Using this value of the spring constant k, together with m = 6, we have dx 6 = 0. + dt2 As in the earlier general discussion, the solution of this equation is X(t) = ( cos(4t) + C2 sin(4t). We are given the initial condition that x(0) = 0.4. But, from the previous equation, x(0) = cz. Therefore cn = . Differentiating, we get x'(t) = -4c sin(4t) + 4c2 cos(4t). Since the initial velocity is given as x'(O) = 0, we have cz = 0 and so the solution is = x(t) = In a certain survey, 511 people chose to respond to this question: "Should passwords be replaced with biometric security (fingerprints, etc)?"Among the respondents, 51% said "yes." We want to test the claim that more than half of the population believes that passwords should bereplaced with biometric security. Complete parts (a) through (d) below.a. Are any of the three requirements vicated? Can a test about a population proportion using the normal approximation method be used?OA. All of the conditions for testing a claim about a population proportion using the normal approximation method are satisfied, so the methodcan be used.B. The conditions np 25 and nq 25 are not satisfied, so a test about a population proportion using the normal approximation method cannotbe used.OC. One of the conditions for a binomial distribution are not satisfied, so a test about a population proportion using the normal approximatingmethod cannot be used.D. The sample observations are not a random sample, so a test about a population proportion using the normal approximating methodcannot be used. Answer all the questions RIGHT and i will give you a brainly 1)The landscaper uses 4 bags of topsoil to cover 3/8 of the garden. How many bags of topsoil will he need to buy to cover the whole garden?2)The road crew was laying down asphalt at a rate of 1 2/3 yards per 1 7/9 minutes. How many yards of asphalt can they lay per minute? (Put your answer in decimal form)3)Maleah turned on the water in the kitchen. For every 1 3/4 minute, 1 2/3 gallons of water went into the sink. How many gallons of water filled the sink per minute?4)James earned $26. 00 last week from mowing lawns for 2 hours. This week he mowed lawns for 4 hours and earned $52. 0. Is the amount of money he earns proportional to the number of hours he works? Yes or No The student council at Newberg High School is making T-shirts to sell for a fundraiser, at a price of $12 apiece. The costs, meanwhile, are $7 per shirt, plus a setup fee of $65. Selling a certain number of shirts will allow the student council to cover their costs. How many shirts must be sold? What will the costs be? What data did the tests on the purfied substance produce? How did this data further support the hypothesis that DNA is the transforming factor? The value of the integral dx/(1-2x^2) is When 1367 J of heat energy is added to 40. 1 g of ethanol, C2H6O, the temperature increases by 13. 9 C. Calculate the molar heat capacity of C2H6O. P= J/(molC) Use the pythagorean Theorem to find the length of a right triangles hypotenuse. The longer sides are 9 cm and 12 cm long In the diagram below, DE is parallel to AB. If CE = 2,AC = 3.6, AB = 4.2, and DC = 2.4, find the length of CB.Figures are not necessarily drawn to scale. Fuel-efficient electric and gas/electric hybrid vehicles produce very little sound at normal driving speeds, and are thus difficult for the visually impaired to detect. Does this raise problems for engineers similar to those raised by roundabouts? In what ways are these problems similar? In what ways are they different? The areas of two triangles are 50 cm2 and 98 cm2. what is the ratio of their perimeters?a ) 25/49b ) 50/98c ) 625/2401d ) 5/7e ) 2500/9604 which of the options is not a general requirement for complex multicellular life? cells must communicate with one another. cells must participate in a network of genetic interactions that regulate cell division. cells must stick together. all of these choices are correct. individual cells must retain a full range of functions, including reproduction. The following table gives the average monthly exchange rate between the us dollar and the australian dollar for 2018. it shows that 1 us dollar was equivalent to 1.256 australian dollars in january 2018. a. evaluate the components of time series of average monthly exchange rate b. smooth out the patterns that includes everything the model learned so far based on history record of the exchange rate. the forecast in the first month was 1.235. you are free to choose the suitable coefficient to conduct the model. explain the decision on the coefficient c. would you apply the method in part (b) to forecast the monthly exchange rate for 2020? please suggest and conduct all possible techniques that may apply to predict monthly foreign exchange rate in year 3. d. compare the forecasting results of different techniques applied in part (c). which ones yield more accurate results? A brick wall be shaped like a rectangular prism.the wall needs to be 3 feet tall, and the builder have enough bricks for the wall to have a volumn of 330 cubic feet. why can you only only photograph a lunar eclipse somtimes A laser beam is aimed through a circular aperture of diameter 1 mm. a. If the laser beam is red with a wavelength of 632. 8 nm, what is the angle from the center of the Airy disk to the first dark ring? (2 points)sin(p) = 632. 8*10^-9 /. 001sin^-1(. 0006328) =. 0363 degreesb. If the screen you are projecting the Airy disk onto is 2 m from the aperture, what is the distance between the center of the disk and the first dark ring? (2 points)Thanks everyone who can help! The null and alternate hypotheses are:H0 : d 0H1 : d > 0The following sample information shows the number of defective units produced on the day shift and the afternoon shift for a sample of 4 days last month. Day: 1, 2, 3, 4Day Shift: 12, 16, 20, 24Afternoon Shift: 12, 10, 16, 18At the 0. 05 significance level, is there a difference in the mean number of citations given by the two shifts?a. What is the p-value? Let g(x.y)= 15x2 +2y2. Compute g(3,3), g(0,-2), and g(a,b). g(3,3)= _____